2022 bc2

You might also like

Download as pdf or txt
Download as pdf or txt
You are on page 1of 21

TI in Focus: AP® Calculus

2022 AP® Calculus Exam: BC2


Scoring Guidelines and Solutions

Stephen Kokoska
Professor Emeritus, Bloomsburg University
Former AP® Calculus Chief Reader

*AP is a registered trademark of the College Board, which was


not involved in the production of, and does not endorse, this product.
BC2 Particle Motion

Outline

(1) Free Response Question

(2) Point Distribution

(3) Solutions (using technology)

(4) Scoring Notes

(5) Common Errors

(6) Problem Extensions


BC2 Particle Motion
AP Calculus AB/BC 2022 Scoring Guidelines VR117060 v1.0

Part A (BC): Graphing calculator required


Question 2 9 points

General Scoring Notes


The model solution is presented using standard mathematical notation.

Answers (numeric or algebraic) need not be simplified. Answers given as a decimal approximation should be
correct to three places after the decimal point. Within each individual free-response question, at most one
point is not earned for inappropriate rounding.

A particle moving along a curve in the xy -plane is at position ( x( t ) , y ( t ) ) at time t > 0. The particle moves
dx dy
in such a way that =
dt dt ( )
1 + t 2 and = ln 2 + t 2 . At time t = 4, the particle is at the point (1, 5 ) .

Model Solution Scoring


(a) Find the slope of the line tangent to the path of the particle at time t = 4.

dy y′( 4 ) ln 18 Answer 1 point


= = = 0.701018
dx t = 4 x′( 4 ) 17

The slope of the line tangent to the path of the particle at time
t = 4 is 0.701.
Scoring notes:
BC2 Particle Motion

Solution
dy y ′ (4) ln(2 + 42 ) ln 18
= = √ = √ = 0.701
dx t=4 x′ (4) 1+4 2 17
t = 4 is 0.701. BC2 Particle Motion

The slope of the line tangent to the path of the particle at time
Scoring
t = 4 is notes:
0.701.
• To earn the point, the setup used to perform the calculation must be evident in the response. The
Scoring notes:
To earn theexamples
• following point, theearn
setup
theused
y′( 4 )
to perform
point: = the
(
ln 2 + 42)
calculation must, be
0.701,
ln 18
or evident
. in the response. The
x′( 4 ) 1 + 422 17
y ( 4)
′ (
ln 2 + )4 ln 18
• following examples
Note: A response withearn
anthe point: equation
incorrect = 0.701,
x′( 4 ) of the form1 “+function
, or= constant
. ”, such as
42 17
y′( t ) ln (18 )
• Note: = , will
withnotan earn the point. However, such a“ response will be eligible foras
any points for
x′( t ) A response
17 incorrect equation of the form function = constant ”, such
y ′( ) errors
t
similar ln ( in
18 ) subsequent parts.
= , will not earn the point. However, such a response will be eligible for any points for
x′( t ) 17
Total for part (a) 1 point
similar errors in subsequent parts.
Total for part (a) 1 point

© 2022 College Board

© 2022 College Board


BC2 Particle Motion

Common Errors
y ′ (4)
(1) Most students recognized the need to find .
x′ (4)

(2) There were some communication errors, especially in equating a function to a


specific value.

(3) Some students presented the reciprocal of the correct answer.


AP Calculus AB/BC 2022 Scoring Guidelines BC2 Particle Motion VR117060 v1.0

(b) Find the speed of the particle at time t = 4, and find the acceleration vector of the particle at time
t = 4.

( x′( 4 ) )2 + ( y′( 4 ) )2 =17 + ( ln 18 )2 =


5.035300 Speed 1 point

The speed of the particle at time t = 4 is 5.035.

4 4 First component of 1 point


a( 4 )
= x′′( 4=
) , y′′( 4 ) = , 0.970143, 0.444444
17 9 acceleration
Second component of 1 point
The acceleration vector of the particle at time t = 4 is acceleration
0.970, 0.444 .

Scoring notes:
• To earn any of these points, the setup used to perform the calculation must be evident in the
response. The following examples earn the first point: ( x′( 4 ) )2 + ( y′( 4 ) )2 =
5.035 or
4 4
17 + ( ln 18 )2 and x′′( 4 ) , y′′( 4 ) = , would earn both the second and third points.
17 9
• The second and third points can be earned independently.
• If the acceleration vector is not presented as an ordered pair, the x - and y -components must be
labeled.
• If the components of the acceleration vector are reversed, the response does not earn either of the
last 2 points.
BC2 Particle Motion

Solution
p p
Speed = [x′ (4)]2 + [y ′ (4)]2 = 17 + (ln 18)2 = 5.035300

 
4 4
a(4) = ⟨x′′ (4), y ′′ (4)⟩ = √ , = ⟨0.970143, 0.444444⟩
17 9
= 4. 0.444 .
t 0.970, 4 4 BC2 Particle MotionFirst component of 1 point
a( 4 ) x′′( 4=) , y′′( 4 ) = , 0.970143, 0.444444
2 2 17 9 2 acceleration
Speed 1 point
( x′( 4 ) )notes:
Scoring + ( y′( 4 ) ) = 17 + ( ln 18 ) = 5.035300
Second component of 1 point
• To earn any of these points, the setup used to perform the calculation must be evident in the
The acceleration vector of the particle at time t = 4 is acceleration
The response.
speed of the Theparticle
following at time
examples the first point: ( x′( 4 ) )2 + ( y′( 4 ) )2 =
is 5.035.
t = 4 earn 5.035 or
0.970, 0.444 .
2 4 4 4 4 First component of points.1 point
+x′′((ln
a( 4 ) 17 notes: , y)′′( 4and
)18
4= ) x= ′′( 4 ) ,, y′′( 4 ) 0.970143,
= , would earn both the second and third
Scoring 17 9 17 90.444444 acceleration
•• To Theearn second any and
of these
thirdpoints,
points the can setup used independently.
be earned to perform the calculation must be evident in the
Second component of 1 point
There
The response.
must
• Ifacceleration The
be
the acceleration following
supporting
vectorvector examples
of theisparticle work. earn
not presented the
(See
at time ast first
last
=an point:
isitem.)
4ordered pair, ) )2 x+acceleration
( x′( 4the y′( 4 )y)-components
-(and
2
= 5.035 or must be
labeled.
0.970,170.444
+ ( ln 18 . )2 and x′′( 4 ) , y′′( 4 ) = 4 , 4 would earn both the second and third points.
17 9
• If the components of the acceleration vector are reversed, the response does not earn either of the
Scoring notes:
The 2second
• last points.and third points can be earned independently.
• To earn any of these points, the setup used to perform the calculation must be evident in the
• If the acceleration vector is not presented as an ordered pair, the x - andt y -components must2tbe
• A response which correctly calculates expressions for both′ x′′( t2) = ′ and y′′( t ) = ,
Theresponse.
labeled.
first and Thethe following
second examples
componentearn the of firstthe
point:
acceleration y1(+4 )t)22 can
( x ( 4 ) ) + (vector = 5.035be orearned
2 + t2
• but If17the
independently.which fails
+components
( ln 18 )2 toand evaluate
of the )both of these expressions
x′′( 4acceleration
4 4
, y′′( 4 ) = vector ,are reversed,
at t =the
would earn
earns only
4, response does
both the second
1 ofnot
the lasteither
andearn
2 points.
of the
third points.
17 9
• last 2 points.
An unsupported acceleration vector earns only 1 of the last 2 points.
• The second and third points can be earned independently. t 2t
• A response which correctly calculates expressions for both x′′( t ) = Total for andpart
y′′((b)
t ) = 3 points
,
• If the acceleration vector is not presented as an ordered pair, the x - and 1 + yt 2-components must t2
2 + be
but which fails to evaluate both of these expressions at t = 4, earns only 1 of the last 2 points.
labeled.
• If
Anthe components
unsupported of the acceleration
acceleration vector
vector earns are1reversed,
only of the lastthe response does not earn either of the
2 points.
lastcommunication
Good 2 points. skills are necessary. Total for part (b) 3 points
t 2t
• A response which correctly calculates expressions for both x′′( t ) = and y′′( t ) = ,
1 + t2 2 + t2
but which fails to evaluate both of these expressions at t = 4, earns only 1 of the last 2 points.
• If the acceleration vector is not presented as an ordered pair, the x - and y -components must be
• To earn any of2 these points, the setup used 4 to perform the calculation must be evident in the
BC2 Particle Motion
labeled. 4
17 + ( ln 18 ) and x′′( 4 ) , y′′( 4 ) = , would earn both the second and third points.
response. The following examples earn the 9 point: ( x′( 4 ) )2 + ( y′( 4 ) )2 =
17first or of the
• If the components of the acceleration vector are reversed, the response does not5.035 earn either
• The 2second
last points.and third points can be earned4 independently.
4
17 + ( ln 18 )2 and x′′( 4 ) , y′′( 4 ) = , would earn both the second and third points.
• If the acceleration vector is not presented17 as an9 ordered pair, the x - andt y -components must2tbe
• A response which correctly calculates expressions for both x′′( t ) = and y′′( t ) = ,
• labeled.
The
a(4) = second
′′
⟨y (4), andxthird
′′
points
(4)⟩ can be earned
= ⟨0.444, 0.970⟩independently. 1 + t2 2 + t2 ? -0
• but which
If the fails to evaluate
components
acceleration of isboth of these
the acceleration
vector expressions
vector
not presented at tpair,
areordered
as an reversed, earns
4,the
=the responseonly
x - and y1-components
does ofnot
theearn
lasteither
2 points.
of be
must the
• last 2 points.
An unsupported
labeled. acceleration vector earns only 1 of the last 2 points.
•• If
A the components
response of the acceleration
which correctly calculatesvector are reversed,
expressions for boththex′′response
t for
does
( t ) = Total not earn
andpart ) = of23tthe
teither
y′′((b) points
,
last 2 points. 1 + t2 2 + t2
but which fails to evaluate both of these expressions at t = 4, earns only t 1 of the last 2 points.
2t
• A response which correctly calculates expressions for both x′′( t ) = and y′′( t ) = ,
• An unsupported acceleration vector earns only 1 of the last 2 points. 1 + t 2 2 + t2
but which fails to evaluate both of these expressions at t = 4, earns only 1 of
Total forthe last(b)
part 2 points.
3 points
• An unsupported acceleration vector earns only 1 of the last 2 points.
Total for part (b) 3 points
a(4) = ⟨0.970, 0.444⟩

© 2022 College Board


BC2 Particle Motion

Common Errors

(1) Equating the speed function with the value of the function at t = 4.

(2) Use of the notation |v| without any indication about how the quantity v was defined.

(3) Parentheses errors that resulted in ambiguous or incorrect expressions.

(4) Many students found the components of the acceleration vector symbolically.
In this case: chain rule errors, power rule errors.

(5) Some responses found the length of the acceleration vector.

(6) Incorrect labels or no labels on vector components.


AP Calculus AB/BC 2022 Scoring Guidelines BC2 Particle Motion VR117060 v1.0

(c) Find the y -coordinate of the particle’s position at time t = 6.

6 Integrand 1 point
∫4 ln ( 2 + t ) dt
2
y ( 6 ) =y ( 4 ) +
Uses y ( 4 ) 1 point

5 + 6.570517 =
= 11.570517 Answer 1 point

The y -coordinate of the particle’s position at time t = 6 is


11.571 (or 11.570 ).
Scoring notes:
• ( )
For the first point, an integrand of ln 2 + t 2 can appear in either an indefinite integral or an
incorrect definite integral.
• A definite integral with incorrect limits is not eligible for the answer point.
• Similarly, an indefinite integral is not eligible for the answer point.
• For the second point, the value for y ( 4 ) must be added to a definite integral. A response that reports
the correct x -coordinate of the particle’s position at time t = 6 as
6
x( 6 ) = x( 4 ) + ∫4 1 + t 2 dt = 11.200 (or 11.201 ) instead of the y -coordinate, earns 2 out of the
3 points.
• A response that earns the first point but not the second can earn the third point with an answer of
6.571 (or 6.570 ).
• If the differential is missing:
BC2 Particle Motion

Solution

Given: y(4) = 5, Need y(6)


Z 6 Z 6
y ′ (t) dt = y(6) − y(4) ⇒ y(6) = y(4) + y ′ (t) dt
4 4

y(6) = 5 + 6.570517 = 11.570517

The y-coordinate of the particle’s position at time t = 6 is 11.571.


Calculus
11.571AB/BC 2022
(or 11.570 ). Scoring Guidelines BC2 Particle Motion VR117060 v1.0
= 5 + 6.570517 = 11.570517 Answer 1 point
) Find the notes:
Scoring y -coordinate of the particle’s position at time t = 6.

• For
The the first point,
-coordinate
y ( 6 ) y=
6
( anparticle’s
of 2the
y ( 4 ) + ∫ ln
integrandposition
)
+ t 2 dt (
of ln 2 at
+ time )
t 2 can
t =appear
6 is in eitherIntegrand
an indefinite integral or an 1 point
4
11.571 (or 11.570
incorrect ). integral.
definite Uses y ( 4 ) 1 point
• A definite
Scoring notes:integral with incorrect limits is not eligible for the answer point.
=5 +want
We 6.570517 =
to see11.570517
the correct integrand. Answer 1 point
• Similarly, an indefinite integral is not eligible 2 for the answer point.
( )
• For the first point, an integrand of ln 2 + t can appear in either an indefinite integral or an
• For the second point, the value for y ( 4 ) must be added to a definite integral. A response that reports
The incorrect
y -coordinate of the
definite particle’s position at time t = 6 is
integral.
the correct x -coordinate of the particle’s position at time t = 6 as
•11.571 (or 11.570
A definite ). with incorrect limits is not eligible for the answer point.
integral6
∫4 1 + t integral
2
x( 6 ) = x( 4an) +indefinite
• Similarly, dt = 11.200 (or 11.201
is not eligible for )the
instead
answer of point.
the y -coordinate, earns 2 out of the
Scoring notes:
A definite
3 points. integral with correct integrand and incorrect limits cannot resolve to the
•• ForFor the
correct
the first
second
answer.
point,
point, an the value for
integrand
Therefore, ( )
( 42)+must
of lnycannot
this
2 be added
can
t earn appear toina either
the answer
definite
anintegral. A integral
indefinite
point.
responseorthat
an reports
• A theresponse
correct xthat earns the of
-coordinate first
thepoint but not
particle’s the second
position cant earn
at time = 6 the
as third point with an answer of
incorrect definite integral.
6.571 (or 6.570 6).
• A x(definite ) + ∫ with
6 ) = x( 4integral 1 + t 2incorrect limits(or
dt = 11.200 11.201
is not ) instead
eligible of answer
for the the y -coordinate,
point. earns 2 out of the
• If the differential4 is missing:
• 3Similarly,
points. an indefinite
6
integral is not eligible for the answer point.
• o A = y ( 6second
Forresponse
the ) that
4
(
∫ lnpoint,
earns )t 2 value
2 + the
the earnspoint
first the yfirst
( )
for but point
4 not theand
must be is eligible
second
added can for the
to aearn the third
definitethird.pointAwith
integral. an answer
response of
that reports
If there
6.571
the are
(or no
correct bounds on
6 ).
6.570
x -coordinate
theparticle’s
of the integral, thenat the
position time response
t = 6 as
is not eligible for the third
o y ( 6=
point. ) ∫ ln6( 2 + t 2 ) + y ( 4 ) does not earn the first point but is eligible for the second and third
• If the differential
4 is missing:
x( 6 ) = x( 4 ) + ∫ 1 + t dt = 11.200 (or 11.201 ) instead of the y -coordinate, earns 2 out of the
2
points in the
6 4presence of the correct answer.
( )
y ( 6 ) ∫ ln 2 +6t 2 earns the first point and is eligible for the third.
3o points.
=
• A response that
4
6 earns4 the
( )
o y ( 6 ) =y ( 4 ) + ∫ ln 2 + t 2 earns the first two points and is eligible for the third.
first point but not the second can earn the third point with an answer of
o6.571y ( 6(or
= ) 6.570 ( )
∫4 ln ).2 + t + y( 4 ) does not earn the first point but is eligible
2
for the second and third
Total for part (c) 3 points
• If thepoints in the presence
differential of the correct answer.
is missing:
6
The incorrect
y -coordinate definite integral.
of the particle’s position at time tBC2 = 6 is Particle Motion
• Similarly, an indefinite integral is not eligible for the answer point.
A definite
•11.571 (or 11.570 integral
). with incorrect limits is not eligible for the answer point.
• For the second point, the value for y ( 4 ) must be added to a definite integral. A response that reports
• Similarly,
Scoring notes:anx -coordinate
indefinite integral is not eligible for the answer point.
the correct of the particle’s position at time t = 6 as
• For the second point, the value for y ( 4 ) must 2 be added to a definite integral. A response that reports
• For x ( 6 the
) = first
x ( 4 )point,
the correct x -coordinate+ ∫4
6 an integrand
1 + t 2 ( )
of ln 2(or
= 11.200
dt the
of particle’s
can appear inofeither
+ t11.201
position)atinstead
time t = the
an indefinite integral or an
6 asy -coordinate, earns 2 out of the
incorrect definite6 integral.
3 points.
x( 6 ) = x( 4 ) + ∫ 1 + t 2 dt = 11.200 (or 11.201 ) instead of the y -coordinate, earns 2 out of the
• A definite integral 4 with incorrect limits is not eligible for the answer point.
• A response that earns the first point but not the second can earn the third point with an answer of
• 3Similarly,
points. an indefinite integral is not eligible for the answer point.
6.571 (or 6.570 ).
• A response
Forthe thedifferential
second thatpoint,
earns the
the value
first point
for ybut( 4 )not
mustthebe
second
addedcan to aearn the third
definite pointAwith
integral. an answer
response of
that reports
•• If is missing:
6.571 (or 6.570 ).
the correct x -coordinate 6 of the particle’s position at time t = 6 as
Z• 6 Ifo the= (
6 ) ∫ ln6is
y (differential ) + t 2 earns the first point and is eligible for the third.
2 missing:
66 ∫4
x(′6 ) = x( 4 ) + 4
1 + t 2 dt = 11.200 (or 11.201 ) instead of the y -coordinate, earns 2 out of the
y
o =
o
(t)
4 3 points.
dt
yy (( 66= ((
)) ∫ ln
= ))
6.571 22
∫44 ln 22 ++ tt +earns y ( 4 )thedoes
firstnot
point
earnand
theisfirst
eligible
pointforbuttheis third.
1-0-1
eligible for the second and third
• A response
points inthat
6 earns the first point but not the second can earn the third point with an answer of
o y ( 6= ) ∫the presence
( ) of the correct answer.
ln 2 + t 2 + y ( 4 ) does not earn the first point but is eligible for the second and third
6.571 (or 6.570 4 ). 6


y( 6 ) =
o points inythe (
If the differential is ∫missing:
( 4 ) presence
+ ln 2of+ the
4
) 2
earns the
t correct first two points and is eligible for the third.
answer.
6
y ( 6 ) ∫ ln ( 2 +4 t 2 )( earns )the first point and is eligible for the third.
o y ( 6 ) =y6( 4 ) + ∫ ln 2 + t 2 earns the first two points and is eligible
Totalforfor
thepart
third.
(c) 3 points
o =
4
6 Total for part (c) 3 points
∫4 ln ( 2 + t ) + y( 4 ) does not earn the first point but is eligible for the second and third
2
o y ( 6=
)
points in the presence of the correct answer.
6
∫4 ln ( 2 + t ) earns the first two points and is eligible for the third.
2
o y ( 6 ) =y ( 4 ) +

Total for part (c) 3 points


BC2 Particle Motion

Common Errors

(1) Presentation of only an indefinite integral.

(2) Failure to add the initial position y(4).


Z
(3) Attempts to find ln(2 + t2 ) dt using integration by parts.

Z 6
(4) CAS results for ln(2 + t2 ) dt
4
AP Calculus AB/BC 2022 Scoring Guidelines BC2 Particle Motion VR117060 v1.0

(d) Find the total distance the particle travels along the curve from time t = 4 to time t = 6.
Integrand 1 point
( dxdt ) + ( dydt )
6 2 2
⌠ dt

⌡4

= 12.136228 Answer 1 point

The total distance the particle travels along the curve from time
t = 4 to time t = 6 is 12.136.
Scoring notes:
Solution
• The first point is earned for presenting the correct integrand in a definite integral.
Z 6p
• To earn the second point, a response must have earned the first point and must present the value
Distance 12.136.
= [x′ (t)]2 + [y ′ (t)]2 dt
4
• An unsupported answer of 12.136 does not earn either point.
= 12.136
Total for part (d) 2 points
Total for question 2 9 points

( ) ( )
dx
t= 4 to time
⌡46 dt 2
dy
+ t = 6 is
dt
dt 12.136. BC2 Particle Motion
Integrand 1 point



( ) ( )
Scoring
dx
dt
notes:
=412.136228
+
dy 2
dt
dt
Answer 1 point
• The first point is earned for presenting the correct integrand in a definite integral.
= 12.136228 Answer 1 point
Z•The□To earn the second point, a response must have earned the first point
total
p distance the particle travels along the curve from time
and must present the value
12.136. ′ (t)]2 + [y ′ (t)]2 dt
t = 4 to[x time t = 6 is 12.136.
The
•□ Antotalunsupported
distance theanswer
particleoftravels
12.136along
doesthe
notcurve from time
earn either point.
Scoring
t = 4 to notes:
time t = 6 is 12.136.
Total for part (d) 2 points
The first
•Scoring notes:point is earned for presenting the correct integrand in a definite integral.
Total for question 2 9 points
•• To Theearnfirstthe second
point point,for
is earned a response must
presenting the have earned
correct the first
integrand in apoint andintegral.
definite must present the value
12.136.
• To earn the second point, a response must have earned the first point and must present the value
• An unsupported answer of 12.136 does not earn either point.
12.136.
• An unsupported answer of 12.136 does not earn either point. Total for part (d) 2 points
Total forfor
Total question
part (d)2 92 points

Total for question 2 9 points


BC2 Particle Motion

Common Errors

(1) Presentation errors: missing parentheses.


s 2
Z 6 
dy
(2) Presentation of the definite integral 1+ dt.
4 dx
Z 6
dy
(3) Presentation of the definite integral: dt
4 dx
BC2 Particle Motion

Additional Questions

(1) Find the x-coordinate of the particle’s position at time t = 6.

(2) Find the particle’s initial position, that is, at time t = 0.

(3) Find the particle’s distance from the origin at time t = 6.

(4) Find the time at which the particle is closest to the origin. What is the position of
the particle at that time? What is the distance?

(5) Find the time t1 at which the particle crosses the x-axis. Find the time t2 at which
the particle crosses the y-axis. Find the total distance the particle travels along the
curve from t = t1 to t = t2 .
BC2 Particle Motion

education.ti.com

You might also like